Difference between revisions of "2020 AMC 10A Problems/Problem 22"

(Solution (Casework))
(Solution (Casework))
Line 28: Line 28:
 
Note that <math>n = 1</math> doesn't work; to prove this, we just have to substitute <math>1</math> for <math>n</math> in the expression.
 
Note that <math>n = 1</math> doesn't work; to prove this, we just have to substitute <math>1</math> for <math>n</math> in the expression.
 
This gives us
 
This gives us
<math></math>\left\lfloor \dfrac{998}{1} \right\rfloor+\left\lfloor \dfrac{999}{1} \right\rfloor+\left\lfloor \dfrac{1000}{1}\right \rfloor<math> = \frac{998}1 + \frac{999}1 + \frac{1000}1 = 2997 = 999 \cdot 3</math> which is divisible by 3.
+
<math>\left\lfloor \dfrac{998}{1} \right\rfloor+\left\lfloor \dfrac{999}{1} \right\rfloor+\left\lfloor \dfrac{1000}{1}\right \rfloor</math> = \frac{998}1 + \frac{999}1 + \frac{1000}1 = 2997 = 999 \cdot 3<math> which is divisible by 3.
Therefore, the case <math>n = 1</math> does not work.
+
Therefore, the case </math>n = 1<math> does not work.
  
  
Line 35: Line 35:
  
  
<b>Case 1:</b> <math>n</math> divides <math>998</math>
+
<b>Case 1:</b> </math>n<math> divides </math>998<math>
  
The first case  does not work, as the three terms in the expression must be <math>(a, a, a)</math>, as mentioned above, so the sum becomes <math>3a</math>, which is divisible by <math>3</math>.
+
The first case  does not work, as the three terms in the expression must be </math>(a, a, a)<math>, as mentioned above, so the sum becomes </math>3a<math>, which is divisible by </math>3<math>.
  
  
<b>Case 2:</b> <math>n</math> divides <math>999</math>
+
<b>Case 2:</b> </math>n<math> divides </math>999<math>
  
Because <math>n</math> divides <math>999</math>, the number of possibilities for <math>n</math> is the same as the number of factors of <math>999</math>, excluding <math>1</math>.
+
Because </math>n<math> divides </math>999<math>, the number of possibilities for </math>n<math> is the same as the number of factors of </math>999<math>, excluding </math>1<math>.
<math>999</math> = <math>3^3 \cdot 37^1</math>
+
</math>999<math> = </math>3^3 \cdot 37^1<math>
So, the total number of factors of <math>999</math> is <math>4 \cdot 2 = 8</math>.
+
So, the total number of factors of </math>999<math> is </math>4 \cdot 2 = 8<math>.
  
However, we have to subtract <math>1</math>, because the case <math>n = 1</math> doesn't work, as mentioned previously.
+
However, we have to subtract </math>1<math>, because the case </math>n = 1<math> doesn't work, as mentioned previously.
<math>8 - 1 = 7</math>
+
</math>8 - 1 = 7<math>
  
 
We now do the same for the third and last case.
 
We now do the same for the third and last case.
  
  
<b>Case 3:</b> <math>n</math> divides <math>1000</math>
+
<b>Case 3:</b> </math>n<math> divides </math>1000<math>
  
Because <math>n</math> divides <math>1000</math>, the number of possibilities for <math>n</math> is the same as the number of factors of <math>1000</math>, excluding <math>1</math>.
+
Because </math>n<math> divides </math>1000<math>, the number of possibilities for </math>n<math> is the same as the number of factors of </math>1000<math>, excluding </math>1<math>.
<math>1000</math> = <math>5^3 \cdot 2^3</math>
+
</math>1000<math> = </math>5^3 \cdot 2^3<math>
So, the total number of factors of <math>1000</math> is <math>4 \cdot 4 = 16</math>.
+
So, the total number of factors of </math>1000<math> is </math>4 \cdot 4 = 16<math>.
  
Again, we have to subtract <math>1</math>, for the reason stated in Case 2.
+
Again, we have to subtract </math>1<math>, for the reason stated in Case 2.
<math>16 - 1 = 15</math>
+
</math>16 - 1 = 15<math>
  
  
Line 65: Line 65:
 
Now that we have counted all of the cases, we add them.
 
Now that we have counted all of the cases, we add them.
  
<math>7 + 15 = 22</math>, so the answer is <math>\boxed{\textbf{(A)}22}</math>.
+
</math>7 + 15 = 22<math>, so the answer is </math>\boxed{\textbf{(A)}22}$.
  
  

Revision as of 22:24, 1 February 2020

Problem

For how many positive integers $n \le 1000$ is\[\left\lfloor \dfrac{998}{n} \right\rfloor+\left\lfloor \dfrac{999}{n} \right\rfloor+\left\lfloor \dfrac{1000}{n}\right \rfloor\]not divisible by $3$? (Recall that $\lfloor x \rfloor$ is the greatest integer less than or equal to $x$.)

$\textbf{(A) } 22 \qquad\textbf{(B) } 23 \qquad\textbf{(C) } 24 \qquad\textbf{(D) } 25 \qquad\textbf{(E) } 26$


Solution (Casework)

Expression: \[\left\lfloor \dfrac{998}{n} \right\rfloor+\left\lfloor \dfrac{999}{n} \right\rfloor+\left\lfloor \dfrac{1000}{n}\right \rfloor\]

Solution:

Let $a = \left\lfloor \frac{998}n \right\rfloor$

Notice that for every integer $n \neq 1$,

$\bullet$ if $\frac{998}n$ is an integer, then the three terms in the expression above must be $(a, a, a)$,

$\bullet$ if $\frac{999}n$ is an integer, then the three terms in the expression above must be $(a, a + 1, a + 1)$, and

$\bullet$ if $\frac{1000}n$ is an integer, then the three terms in the expression above must be $(a, a, a + 1)$.

This is due to the fact that $998$, $999$, and $1000$ share no common factors collectively (other than 1).


Note that $n = 1$ doesn't work; to prove this, we just have to substitute $1$ for $n$ in the expression. This gives us $\left\lfloor \dfrac{998}{1} \right\rfloor+\left\lfloor \dfrac{999}{1} \right\rfloor+\left\lfloor \dfrac{1000}{1}\right \rfloor$ = \frac{998}1 + \frac{999}1 + \frac{1000}1 = 2997 = 999 \cdot 3$which is divisible by 3. Therefore, the case$n = 1$does not work.


Now, we test the three cases mentioned above.


<b>Case 1:</b>$ (Error compiling LaTeX. Unknown error_msg)n$divides$998$The first case  does not work, as the three terms in the expression must be$(a, a, a)$, as mentioned above, so the sum becomes$3a$, which is divisible by$3$.


<b>Case 2:</b>$ (Error compiling LaTeX. Unknown error_msg)n$divides$999$Because$n$divides$999$, the number of possibilities for$n$is the same as the number of factors of$999$, excluding$1$.$999$=$3^3 \cdot 37^1$So, the total number of factors of$999$is$4 \cdot 2 = 8$.

However, we have to subtract$ (Error compiling LaTeX. Unknown error_msg)1$, because the case$n = 1$doesn't work, as mentioned previously.$8 - 1 = 7$We now do the same for the third and last case.


<b>Case 3:</b>$ (Error compiling LaTeX. Unknown error_msg)n$divides$1000$Because$n$divides$1000$, the number of possibilities for$n$is the same as the number of factors of$1000$, excluding$1$.$1000$=$5^3 \cdot 2^3$So, the total number of factors of$1000$is$4 \cdot 4 = 16$.

Again, we have to subtract$ (Error compiling LaTeX. Unknown error_msg)1$, for the reason stated in Case 2.$16 - 1 = 15$Now that we have counted all of the cases, we add them.$7 + 15 = 22$, so the answer is$\boxed{\textbf{(A)}22}$.


~dragonchomper

Video Solution

https://youtu.be/Ozp3k2464u4

~IceMatrix

See Also

2020 AMC 10A (ProblemsAnswer KeyResources)
Preceded by
Problem 21
Followed by
Problem 23
1 2 3 4 5 6 7 8 9 10 11 12 13 14 15 16 17 18 19 20 21 22 23 24 25
All AMC 10 Problems and Solutions

The problems on this page are copyrighted by the Mathematical Association of America's American Mathematics Competitions. AMC logo.png